ISL 85 - disuguaglianza (non troppo!) facilina

Polinomi, disuguaglianze, numeri complessi, ...
Rispondi
ma_go
Site Admin
Messaggi: 1906
Iscritto il: 01 gen 1970, 01:00

ISL 85 - disuguaglianza (non troppo!) facilina

Messaggio da ma_go »

siano $ x_1, x_2, ..., x_n $ reali positivi.
as usual, adottiamo la convenzione che $ x_{n+i} = x_i $.
dimostrare che $ \displaystyle \sum_{i = 1}^{n} \frac{x_i^2}{x_i^2 + x_{i+1}x_{i+2}} \le n-1 $

edit: avevo completamente lisciato la soluzione :)
Ultima modifica di ma_go il 26 lug 2005, 17:33, modificato 1 volta in totale.
Igor
Messaggi: 108
Iscritto il: 01 gen 1970, 01:00

Messaggio da Igor »

Dimostriamolo per induzione su $ n $.

Per $ n=2 $, abbiamo due numeri: $ x_1,x_2 $

La disuguaglianza diventa

$ \frac{x_1^2}{x_1^2+x_1x_2}+\frac{x_2^2}{x_1^2+x_1x_2}\leq 1 $.

Svolgendo i calcoli ci resta

$ 0\leq 0 $ che è sempre vera.

Supponiamo ora che la disuguaglianza sia vera per un certo $ n $, cioè che dati $ x_1,x_2,\ldots ,x_n $ reali positivi, si abbia

$ P\leq n-1 $, dove con $ P $ abbiamo indicato il termine di sinistra della disuguaglianza di partenza.Dobbiamo allora dimostrare che,dati $ x_1,x_2,\ldots ,x_n,x_{n+1} $ si abbia

$ P+\frac{x_{n+1}^2}{x_{n+1}^2+x_1x_2}\leq n $

$ n-1+\frac{x_{n+1}^2}{x_{n+1}^2+x_1x_2}\leq n $ per ipotesi induttiva

Svolgendo i conti resti

$ x_1x_2\geq 0 $ che è sempre vera poichè si tratta di numeri positivi.

La disuguaglianza è dunque verificata per induzione.
ma_go
Site Admin
Messaggi: 1906
Iscritto il: 01 gen 1970, 01:00

Messaggio da ma_go »

uhm, no:
c'è qualche piccolo problema..
l'ipotesi induttiva non salta fuori così facilmente: i primi n termini del membro di sinistra *non sono* P: gli ultimi 2 termini sono diversi (check it!)!!
pregevole, comunque... magari si riconduce davvero ad una cosa con 3 termini anziché con n :P
Avatar utente
Boll
Messaggi: 1076
Iscritto il: 01 gen 1970, 01:00
Località: Piacenza

Re: ISL 85 - disuguaglianza (non troppo!) facilina

Messaggio da Boll »

Proviamo un pò...
ma_go ha scritto:siano $ x_1, x_2, ..., x_n $ reali positivi.
as usual, adottiamo la convenzione che $ x_{n+i} = x_i $.
dimostrare che $ \displaystyle \sum_{i = 1}^{n} \frac{x_i^2}{x_i^2 + x_{i+1}x_{i+2}} \le n-1 $
Poichè


$ \dfrac{x_i^2}{x_i^2+x_{i+1}x_{i+2}}=1-\dfrac{x_{i+1}x_{i+2}}{x_i^2+x_{i+1}x_{i+2}} $

Possiamo riscrivere la tesi come

$ \displaystyle \sum_{i = 1}^{n} \frac{x_{i+1}x_{i+2}}{x_i^2 + x_{i+1}x_{i+2}} \ge 1 $

Per Cauchy avremo

$ \displaystyle \left( \sum_{i=1}^n x_{i+1}x_{i+2}(x_i^2+x_{i+1}x_{i+2})\right)\ \left( \sum_{i = 1}^{n} \frac{x_{i+1}x_{i+2}}{x_i^2 + x_{i+1}x_{i+2}}\right) $$ \displaystyle\ge \left( \sum_{i=1}^n x_{i+1}x_{i+2}\right)^2 $

Rimane da provare che

$ \displaystyle \left(\sum_{i=1}^n x_{i+1}x_{i+2}\right)^2\ge \sum_{i=1}^n x_{i+1}x_{i+2}(x_i^2+x_{i+1}x_{i+2}) $


che è ovvia (facendo i conti a sinistra rimane una somma di addendi positivi, a destra 0)

Al solito, poichè tutti i passaggi svolti sono invertibili, se sono corretti, la tesi è dimostrata

EDIT: Corretta una stronzata che però non inficiava niente
Ultima modifica di Boll il 27 lug 2005, 13:14, modificato 2 volte in totale.
Simo_the_wolf
Moderatore
Messaggi: 1053
Iscritto il: 01 gen 1970, 01:00
Località: Pescara

Messaggio da Simo_the_wolf »

Facciamola un po' piu' semplice...

Diciamo che $ x_a $ e $ x_b $ sono rispettivamente il minimi della successione con $ x_a\leq x_b $. Abbiamo che, essendo ogni termine della sommatoria e' minore di 1:
$ \displaystyle \sum_{i=1}^n \frac {x_i^2}{x_i^2+x_{i+1}x_{i+2}} \leq $$ \displaystyle n-2+\frac {x_a^2}{x_a^2+x_{a+1}x_{a+2}}+\frac {x_b^2}{x_b^2+x_{b+1}x_{b+2}} \leq $
$ \displaystyle \leq n-2 + \frac {x_a^2}{x_a^2+x_b^2}+\frac {x_b^2}{x_b^2+x_a^2} =n-1 $

Abbiamo usato il fatto che ogni numero della successione e' maggiore o uguale a $ x_a $ e tutti i numeri eccetto $ x_a $ sono maggiori o uguali a $ x_b $. La dimostrazione salterebbe se $ n=2 $ ma in quel caso e' stato gia' dimostrato... :D Byez!!
Rispondi